0% found this document useful (0 votes)
244 views7 pages

Assignment4 Solutions Chap6

1. The document provides solutions to assignment problems related to digital communication systems operating over AWGN channels. 2. Problem 1 calculates the probability of bit error for coherent detection of binary FSK signals. Problem 2 finds the optimum detection threshold for a correlator receiver detecting BPSK signals. 3. Problem 3 calculates the minimum probability of bit error and probability of bit error for a given threshold for BPSK signals received over an AWGN channel. 4. Problem 4 calculates the probability of bit error for an 8-ary orthogonal MFSK system transmitted over an AWGN channel.

Uploaded by

Mohit soni
Copyright
© © All Rights Reserved
We take content rights seriously. If you suspect this is your content, claim it here.
Available Formats
Download as PDF, TXT or read online on Scribd
0% found this document useful (0 votes)
244 views7 pages

Assignment4 Solutions Chap6

1. The document provides solutions to assignment problems related to digital communication systems operating over AWGN channels. 2. Problem 1 calculates the probability of bit error for coherent detection of binary FSK signals. Problem 2 finds the optimum detection threshold for a correlator receiver detecting BPSK signals. 3. Problem 3 calculates the minimum probability of bit error and probability of bit error for a given threshold for BPSK signals received over an AWGN channel. 4. Problem 4 calculates the probability of bit error for an 8-ary orthogonal MFSK system transmitted over an AWGN channel.

Uploaded by

Mohit soni
Copyright
© © All Rights Reserved
We take content rights seriously. If you suspect this is your content, claim it here.
Available Formats
Download as PDF, TXT or read online on Scribd
You are on page 1/ 7

Assignment 4 Solutions

1. Sklar, 4.7
Find the probability of bit error, PB , for the coherent matched filter detection of the equally
likely binary FSK signals
s1 (t) = 0.5 cos 2000πt

and
s2 (t) = 0.5 cos 2020πt

where the two-sided AWGN power spectral density is N0 /2 = 0.0001. Assume that the symbol
duration is T = 0.01 s.

Solutions: The bit energy is

0.52
Eb = ST = (0.01) = 0.00125 Joule
2

The correlation coefficient is


∫ T ∫ T
1 1
ρ = s1 (t)s2 (t) dt = 0.5 cos(2π 1000t) 0.5 cos(2π 1010t) dt
Eb 0 Eb 0
∫ 0.01
0.25 1
= [cos(2π 10t) + cos(2π 2010t)] dt
0.00125 0 2
[ ]0.01
sin(2π 10t) sin(2π 2010t)
= 100 +
20π 4020π 0
= 0.935 + 0.005 = 0.94

Then √  (√ )
Eb (1 − ρ)  0.00125(0.06)
Pb = Q  =Q = Q(0.612) = 0.27
N0 0.0002

This error rate is much greater than if the tone spacing required for coherent orthogonal sig-
1
nalling, 2T = 50 Hz had been used, instead of the 10 Hz specified.

1
2. Sklar, 4.8
Find the optimum (minimum probability of error) threshold γ0 , for detecting the equally likely
√ √
signal s1 (t) = 2E/T cos ω0 t and s2 (t) = E/2T cos(ω0 t + π) in AWGN, using a correlator

receiver. Assume a reference signal of ψ1 (t) = 2/T cos ω0 t

Solutions: Signal s1 (t) and s2 (t) can be represented as



s1 (t) = Eψ1 (t)
1√
s2 (t) = − Eψ1 (t)
2

With following conditional PDF, then


√ 1√
µ1 = E µ2 = − E
2

The decision threshold V is


√ ( √ )
E + − 12 E √
µ1 + µ2 E
V = = =
2 2 4

fU (u|′ 0′ ) fU (u|′ 1′ )

P (x|′ 1′ )

P (x|′ 0′ )

µ2 x µ1 U = y(T )
V

2
3. Sklar, 4.9

A system using matched filter detection of equally likely BPSK signal, s1 (t) = 2E/T cos ω0 t

and s2 (t)) = 2E/T cos(ω0 t + π), operates in AWGN with a received Eb /N0 of 6.8 dB. Assume

that E[z(T )] = ± E.

(a) Find the minimum probability of bit error, PB , for this signal set and Eb /N0 .

(b) If the decision threshold is V = 0.1 E, find PB .

Solutions: (a) Since (√ )


2Eb
PB = Q
N0
where
Eb
= 6.8 dB = 4.786
N0
Therefore,
(√ )
PB = Q 2 × 4.786 = Q(3.09) = 10−3

(b) The bit error rate can be derived as


∫ √ ∫
1 0.1 E
1 ∞
PB = f (u|s1 )du + f (u|s2 )du
2 −∞ 2 0.1√E
 ( √ )2   ( √ )2 
∫ 0.1√E ∫ ∞
1 1 1 u− E  1 1 1 u+ E 
= √ exp − du + √ exp − du
2 −∞ σ0 2π 2 σ0 2 0.1√E σ0 2π 2 σ0

Let √
u− E
a1 = ; σ0 da1 = du
σ0
and √
u+ E
a2 = ; σ0 da2 = du
σ0
Then
∫ √
− 0.9σ [ 2] ∫ [ 2]
1 ∞
E
1 0 1 a1 1 a
PB = √ exp − da1 + √ √ exp − 2 da2
2 −∞ 2π 2 2 1.1σ E σ0 2π 2
0

For the symmetrical Gaussian function,


∫ −x [ 2] ∫ ∞ [ 2]
u u
exp − du = exp − du
−∞ 2 x 2

BER is ( √ ) ( √ )
1 0.9 E 1 1.1 E
PB = Q σ0 + Q σ0
2 2

3
For binary matched filter detection, we can write E = Eb and σ02 = N0 /2, then
( √ ) ( √ )
1 2Eb 1 2Eb
PB = Q 0.9 + Q 1.1
2 N0 2 N0
1 ( √ ) 1 ( √ )
= Q 0.9 2 × 4.786 + Q 1.1 2 × 4.786
2 2
1 1
= Q(0.9 × 3.09) + Q(1.1 × 3.09)
2 2
1 1 1
= Q(2.78) + Q(3.4) = (0.0027 + 0.0003) = 1.4 × 10−3
2 2 2

fU (u|s2 ) fU (u|s1 )

P (x|s1 )

P (x|s2 )
√ √
− E x E U = y(T )

V = 0.1 E

4. Sklar, 4.13
Consider a coherent orthogonal MFSK system with M = 8 having the equally likely waveforms
si (t) = A cos 2πfi t, i = 1, · · · , M, 0 ≤ t ≤ T , where T = 0.2 ms. The received carrier amplitude,
A, is 1 mV, and the two-sided AWGN spectral density, N0 /2, is 10−11 W/Hz. Calculate the
probability of bit error, PB .

Solution: We have (√ )
Es
PE = (m − 1)Q
N0
where
A2 (10−3 )2
Es = T = × 0.2 × 10−3 = 10−10
2 2
and (√ )
10−10
PE = (8 − 1)Q = 7Q(2.236) = 7 × 0.0127 = 8.89 × 10−2
2 × 10−11

2k−1 22 4
PB = PE = 3 PE = PE = 5 × 10−2
2 −1
k 2 −1 7

4
5. Sklar 3.1 Determine whether or not s1 (t) and s1 (t) are orthogonal over the interval (−1.5T2 <
t < 1.5T2 ), where s1 (t) = cos(2πf1 t + ϕ1 ), s2 (t) = cos(2πf2 t + ϕ2 ) and f2 = 1/T2 for the
following cases

(a) f1 = f2 and ϕ1 = ϕ2

(b) f1 = 13 f2 and ϕ1 = ϕ2

(c) f1 = 2f2 and ϕ1 = ϕ2

(d) f1 = πf2 and ϕ1 = ϕ2

(e) f1 = f2 and ϕ1 = ϕ2 + π/2

(f) f1 = f2 and ϕ1 = ϕ2 + π

Solutions: (a) f1 = f2 and ϕ1 = ϕ2


∫ 1.5T2 ∫ 1.5T2
s1 (t) · s2 (t)dt = s21 (t)dt ̸= 0
−1.5T2 −1.5T2

therefore, not orthogonal.

(b) f1 = 31 f2 and ϕ1 = ϕ2
Let ϕ1 = ϕ2 = 0,
∫ 1.5T2 ∫ 1.5T2 ) ∫ ( ( )
1 2 1 1.5T2 4
s1 (t) · s2 (t)dt = cos 2π f2 tdt + cos 2π f2 tdt (1)
−1.5T2 2 −1.5T2 3 2 −1.5T2 3
sin 2π sin 4π
= + =0 (2)
4/3π(1/T2 ) 8/3π(1/T2 )

therefore, orthogonal.

(c) f1 = 2f2 and ϕ1 = ϕ2 Let ϕ1 = ϕ2 = 0,


∫ 1.5T2 ∫ 1.5T2
1
s1 (t) · s2 (t)dt = (cos 2πf2 t + cos 6πf2 t)dt = 0
−1.5T2 2 −1.5T2

therefore, orthogonal.

(d) f1 = πf2 and ϕ1 = ϕ2 Let ϕ1 = ϕ2 = 0,


∫ 1.5T2 ∫ b ∫ b
1 1
s1 (t) · s2 (t)dt = cos(π − 1)2πf2 tdt + cos(π + 1)2πf2 tdt ̸= 0
−1.5T2 2 a 2 a

therefore, not orthogonal.

(e) f1 = f2 and ϕ1 = ϕ2 + π/2


∫ b
sin 2πf2 t · cos 2πf2 tdt = 0
a

therefore, orthogonal.

5
(f) f1 = f2 and ϕ1 = ϕ2 + π Let ϕ1 = 0,
∫ b
cos2 (2πf2 t) dt ̸= 0
a

not orthogonal.

6. Sklar, 3.2:

(a) Show that the three functions illustrated in figure below are pairwise orthogonal over the
interval (-2,2).

(b) Determine the value of the constant A, that makes the set of functions in part (a) an
orthonormal set.

(c) Express the following waveform, x(t), in terms of the orthonormal set of part (b)

 1 for 0 ≤ t ≤ 2
x(t) =
 0 otherwise

ψ1 (t) ψ2 (t) ψ3 (t)


A
A

-2 2 -2 -2 -1 1 2
t t t
-1 0 -1 2 0

-A -A
-A

Solution (a):
∫ 2 ∫ −1 ∫ 0 ∫ 1 ∫ 2
ψ1 (t)ψ2 (t)dt = (−A)(−A)dt + (A)(−A)dt + (A)(A)dt + (−A)(A)dt
−2 −2 −1 0 1
= A − A2 + A2 − A2 = 0
2

∫ 2 ∫ −1 ∫ 0 ∫ 1 ∫ 2
ψ1 (t)ψ3 (t)dt = (−A)(−A)dt + (A)(−A)dt + (A)(−A)dt + (3)
(−A)(−A)dt
−2 −2 −1 0 1
= A − A2 − A2 + A2 = 0
2

∫ 2 ∫ 0 ∫ 2
ψ2 (t)ψ3 (t)dt = (−A)(−A)dt + (A)(−A)dt = 2A2 − 2A2 = 0
−2 −2 0

(b):
∫ 2 ∫ 0
ψ32 (t)dt = A2 dt = 2A2 + 2A2 = 4A2
−2 −2

To be orthonormal, 4A2 = 1 which implies A = 1/2.

(c): x(t) = ψ2 (t) − ψ3 (t)

6
7. Sklar 3.3 Consider the functions

ψ1 (t) = exp(−|t|) ψ2 (t) = 1 − A exp(−2|t|)

Determine the constant, A, such that ψ1 (t) and ψ2 (t) are orthogonal over the interval (−∞, ∞).

Solution: the correlation between ψ1 (t) and ψ2 (t) is


∫ 0 ∫ ∞
R = et (1 − Ae2t ) dt + e−t (1 − Ae−2t ) dt
−∞ 0
∫ 0 ∫ ∞
= (et − Ae3t ) dt + (e−t − Ae−3t ) dt
−∞ 0
[ ]0 [ ]∞
Ae3t Ae−3t A A 2A
= et − + −e−t + =1− − [−1 + ] = 2 −
3 −∞ 3 0 3 3 3

In order to make ψ1 (t) and ψ2 (t) orthogonal, we need R = 0, which solves A = 3.

You might also like